Đến nội dung

Hình ảnh

Đa thức đối xứng và chứng minh cho một số bất đẳng thức hoán vị.

* - - - - 1 Bình chọn

  • Please log in to reply
Chủ đề này có 32 trả lời

#1
10maths_tp0609

10maths_tp0609

    Zarai Nakeda XIII

  • Thành viên
  • 218 Bài viết
I. Các kiến thức :
_ Với mỗi bộ 3 số $ a , b,c$ ta kí hiệu
$p = a+b+c$
$ q= ab+bc+ca$
$r= abc$
Ta có một số đẳng thức sau :

$\sum \limits_{sym} a^2=p^2-2q$

$\sum \limits_{sym} a^3 = p^3 - 3pq +3r$

$\sum \limits_{sym} a^2(b+c) = pq - 3r $

$\sum \limits_{sym} a^4 = p^4 + 2p^2 + 4pr - 4p^2q$

$\sum \limits_{sym} b^2c^2 = q^2 - 2pr $

$\sum \limits_{sym} a^3(b+c) = p^2q - 2q^2 - pr$

$ (a-b)^2(b-c)^2(c-a)^2= p^2q^2 + 18pqr - 27 r^2 - 4q^3 - 4p^3r$

_ Hàm số $f(X) = AX^2 + BX +C$ ( $A>0 $) $ X_{ct} = \dfrac{-B}{2A} $
+) $ \geq 0 \forall X \Leftrightarrow \Delta =B^2 - 4AC \leq 0 $
+)$ \geq 0 \forall X \geq 0 \Leftrightarrow \large\left[{\left\{\begin{array}{l}0\geq X_{ct} (\text{hoac} B\geq 0 ) \\f(0)\geq 0 ( \text{hoac} C \ge 0) \end{array}\right. \\{ \left\{\begin{array}{l}X_{ct}\geq 0 ( \text{hoac} B \le 0) \\f(X_{ct})\geq 0(\text{hoac} \Delta = B^2 - 4 AC \leq 0) \end{array}\right. $

II . Các ví dụ :
Ví dụ 1 :
Cho $a,b,c \geq 0 $ thỏa mãn :$a+b+c= 3$
CM $ a^2b+b^2c+c^2a \leq 4 :Rightarrow$
Giải :
$:Rightarrow\Leftrightarrow 2\sum \limits_{cyc} a^2b \leq 8 $
$\Leftrightarrow (\sum \limits_{cyc} a^2b + \sum \limits_{cyc}ab^2 ) + (\sum \limits_{cyc}a^2b - \sum \limits_{cyc}ab^2) \leq 8$
$ \Leftrightarrow \sum \limits_{sym}a^2(b+c) + (a-b)(b-c)(c-a) \leq 8$
Rõ ràng ta chỉ cần CM bđt trong TH $(a-b)(b-c)(a-c) \geq 0 $:
$\sum \limits_{sym}a^2(b+c) + \sqrt{(a-b)^2(b-c)^2(c-a)^2} \leq 8 $

$\Leftrightarrow (pq - 3r) + \sqrt {(p^2q^2+ 18 pqr - 27 r^2 - 4q^3 - 4q^3r) } \leq 8 $

$\Leftrightarrow (p^2q^2 + 18pqr - 27r^2 - 4q^3 - 4p^3r) \leq (8-pq +3r)^2$

$\Leftrightarrow 36r^2 + (4p^3 - 24pq +48)r +4q^3 -16pq +64 \geq 0 $

$ \Leftrightarrow 9r^2 + (p^3-6pq +12 ) r + q^3 - 4pq +16 \geq 0 $
Thay $p = 3$ :
$ 9r^2 +(39-18q)r +q^3 -12q +16 \geq 0 $
Đặt $ f( r) = 9r^2 +(39-18q)r +q^3 -12q +16$
$r_{ct} = \dfrac{-39+18q}{18} $
Xét 2 TH :
1) $ 0 \leq q \leq \dfrac{39}{18} \Rightarrow r_{ct} \leq 0 $
$ f(0) = q^3 - 12q +16 = (q+4)(q-2)^2 \geq 0 $
2) $ \dfrac{39}{18} \leq q \leq 3 \Rightarrow r_{ct} \geq 0 $
$ f(r_{ct}) = 24q^3 - 216q^2 + 648q - 630 \geq 0 \forall q \in [\dfrac{39}{18} ; 3 ] $
Vậy $ f( r) \geq 0 \forall r \geq 0 \Rightarrow $ bđt được CM

Ví dụ 2 :
Cho $a,b,c $ là các số ko âm thỏa mãn : $a+b+c = 3$
CMR : $ a^2b +b^2c+ c^2a + 2 ( ab^2 +bc^2 +ca^2 ) \leq 6\sqrt{3}(2)$
Giải :
$ ( 2)\Leftrightarrow 2\sum \limits_{cyc}a^2b + 4\sum \limits_{cyc}ab^2 \leq 12\sqrt{3} $

$ \Leftrightarrow 3\sum \limits_{sym}a^2(b+c) + (\sum \limits_{cyc} ab^2 -\sum \limits_{cyc} a^2b) \leq 12\sqrt{3} $

$ \Leftrightarrow 3\sum \limits_{cyc}a^2(b+c) + (a-b)(b-c)(c-a) \leq 12\sqrt{3} $
Ta chỉ cần CM trong TH :$(a-b)(b-c)(c-a) \geq 0 $

$ 3\sum \limits_{sym}a^2(b+c) + \sqrt{(a-b)^2(b-c)^2(c-a)^2} \leq 12\sqrt{3}$

$\Leftrightarrow 3(pq -3r ) + \sqrt{p^2q^2 +18pqr -27r^2 -4q^3 - 4p^3r} \leq 12\sqrt{3}$

$ \Leftrightarrow p^2q^2 +18pqr - 27r^2 - 4q^3 - 4p^3r \leq (12\sqrt{3}-3pq+9r)^2 $

$ \Leftrightarrow f( r) = 108r^2 + (4p^3-72pq +216\sqrt{3} )r + 4q^3 + 8p^2q^2 -72\sqrt{3}pq +432 \geq 0 $

$ r_{ct} = \dfrac{216q - 108 -216\sqrt{3} }{108}$
Xét 2 TH :
1) $ 0 \leq q \leq \dfrac{216\sqrt{3} +108 }{216} \Rightarrow r_{ct} \leq 0 $

$ f(0) = 4( q+ 12 + 6\sqrt{3} )( q+3 -\sqrt{3})^2 \geq 0 $
2) $ \dfrac{216\sqrt{3} +108 }{216} \leq q \leq 3 \Rightarrow r_{ct} \geq 0 $
$ f(r_{ct}) = 4q^3 -36q^2 +108q +81 - 108\sqrt{3} \geq 0 \forall q \in [\sqrt{3} + \dfrac{1}{2} ; 3] $
Bđt được chứng minh .

Ví dụ 3 :
Tìm hằng số tốt nhất cho bđt sau :
$ k(a+b+c)^4 \geq (a^3b+b^3c+c^3a) + ( a^2b^2 + b^2c^2 + c^2a^2 ) + abc(a+b+c) $
Giải :
Cho $ a= 2, b=1 , c= 0 \Rightarrow k\geq \dfrac{4}{27}$
Ta Cm đây là giá trị cần tìm , nghĩa là :
$ \dfrac{4}{27}(a+b+c)^4 \geq \sum \limits_{cyc}a^3b + \sum \limits_{sym}b^2c^2 + abc\sum \limits_{sym} a $

$\Leftrightarrow \dfrac{8}{27}(a+b+c)^4 \geq (\sum \limits_{cyc}a^3b + \sum \limits_{cyc}ab^3) + 2\sum \limits_{sym}b^2c^2 + (\sum \limits_{cyc}a^3b -\sum \limits_{cyc}ab^3) +2abc(a+b+c) $

$ \Leftrightarrow \dfrac{8}{27}(a+b+c)^4 \geq \sum \limits_{sym}a^3(b+c) +2\sum \limits_{sym}b^2c^2 +(a+b+c)(a-b)(b-c)(a-c) +2abc(a+b+c) $

Chỉ cần Cm trong trường hợp $(a-b)(b-c)(a-c) \geq 0 $
$ \Leftrightarrow \dfrac{8}{27}(a+b+c)^4 \geq p^2q -2q^2- pr +2q^2- 4pr+2pr +p\sqrt{p^2q^2+18pqr-27r^2-4q^3-4p^3r}$

$ \Leftrightarrow p^2(p^2q^2 + 18pqr - 27r^2 - 4q^3- 4p^3r ) \leq [\dfrac{8}{27}p^4 -p^2q+3pr]^2$

$\Leftrightarrow 36p^2r^2 + (\dfrac{52}{9}p^5 -24p^3q)r + \dfrac{64}{729}p^3 +4p^2q^3 - \dfrac{16}{27}p^6q \geq 0 $

$\Leftrightarrow 324p^2 +(1404-648q)r +36q^3-432q +576 \geq 0 $ (chuẩn hóa $p=3$ )
$f( r) =36[9r^2 + (39-18q)r +q^3 -12q+16]$
Xét 2 TH :
1) $ 0 \leq q \leq \dfrac{13}{6} \rightarrow 39-18q \geq 0$
$ f(0)=36(q+4)(q-2)^2 \geq 0 $

2)$ \dfrac{13}{6} \leq q \leq 3 \rightarrow \Delta = (39-18q)^2 -4.9.(q^3-12q+16) = -36q^3 +324q^2 -972q +945 \leq 0 $ VớI $ q \in [\dfrac{13}{6};3]$

Bđt được chứng minh


Ví dụ 4
CM bđt sau vớIi mọi số thực x,y,z :
$ (x^2+y^2+z^2)^2 \geq 3 (x^3y +y^3z+z^3x )(2)$

GiảI :
Nhận xét : Nhân 2 vế bđt $( 2 )$ vớI $ \dfrac{3^4}{(x+y+z)^4} \geq 0 $ và đặt
$ a= \dfrac{3x}{x+y+z} ; b= \dfrac{3y}{x+y+z}; c= \dfrac{3z}{x+y+z}$
Bđt trở thành :
$ (a^2+b^2+c^2)^2 \geq 3 ( a^3b+b^3c+c^3a) $ vớI $a+b+c=3$

$ \Leftrightarrow 2(a^2+b^2+c^2)^2 \geq 3(\sum \limits_{cyc}a^3b +\sum \limits_{cyc}ab^3 ) + 3(\sum \limits_{cyc}a^3b -\sum \limits_{cyc}ab^3 )$

$ \Leftrightarrow 2(a^2+b^2+c^2)^2 \geq 3\sum \limits_{sym}a^3(b+c) + 3(a+b+c)(a-b)(b-c)(a-c)$
Ta chỉ cần CM trong trường hợp $(a-b)(b-c)(a-c) \geq 0 $
$\Leftrightarrow 2(\sum \limits_{sym}a^2)^2 \geq 3\sum \limits_{sym}a^3(b+c) + 3\sum \limits_{sym}a\sqrt{(a-b)^2(b-c)^2(c-a)^2}$

$ \Leftrightarrow 2(p^2-2q)^2 \geq 3(p^2q-2q^2 -pr) + 3p\sqrt{p^sq^2+18pqr -27r^2-4q^3-4p^3r}$

$\Leftrightarrow 9p^2(p^2q^2+18pqr-27r^2-4q^3-4p^3r) \leq [2(p^2-2q)^2-3(p^2q-2q^2-pr)]^2 $

$\Leftrightarrow 252p^2r^2 +(84pq^2-228p^3q+48p^5)r +4p^8-44p^4q+168p^4q^2-272p^2q^3+196q^4 \geq 0 $

$\Leftrightarrow f( r)= 2268r^2 +(252q^2-6156q+11664)r+196q^4-2448q^3+13608q^2-32076q+26244 \geq 0 $

$ \Delta $’$ =(126q^2-3078q+5832)^2-2268(196q^4-2448q^3+13608q^2-32076q+26244) = -8748(7q-18)^2(q-3)^2 \leq 0 $

Bđt được CM

Ví dụ 5 :
Tìm hằng số k tốt nhất cho bđt sau đúng $\forall a,b,c>0$:
$\dfrac{a}{b} +\dfrac{b}{c} +\dfrac{c}{a} +k\dfrac{ab+bc+ca}{a^2+b^2+c^2} \geq 3+k(5)$
GiảI :
Ta có :
$2\sum \limits_{cyc}\dfrac{a}{b}= (\sum \limits_{cyc}\dfrac{a}{b} + \sum \limits_{cyc}\dfrac{b}{a}) + (\sum \limits_{cyc}\dfrac{a}{b} - \sum \limits_{cyc}\dfrac{b}{a}) = \dfrac{\sum \limits_{sym}a^2(b+c)}{abc} + \dfrac{(a-b)(b-c)(c-a)}{abc}$

$ (5)\Leftrightarrow \sum \limits_{cyc}\dfrac{a}{b} + 2k\dfrac{ab+bc+ca}{a^2+b^2+c^2} \geq 6+2k $
$ \Leftrightarrow \dfrac{\sum \limits_{sym}a^2(b+c)}{abc}+ 2k\dfrac{ab+bc+ca}{a^2+b^2+c^2}\geq 6+2k + \dfrac{(a-b)(b-c)(a-c)}{abc}$

Ta chỉ cần CM trong TH $(a-b)(b-c)(a-c) \geq 0 $
$\Leftrightarrow \dfrac{pq-3r}{r} + \dfrac{2kq}{p^2-2q} \geq 6+ 2k+\dfrac{\sqrt{p^2q^2+18pqr-27r^2-4q^3-4p^3r}}{r}$

$ (p^2q^2+18pqr-27r^2-4q^3-4p^3q)(p^2-2q)^2 \leq [(pq-3r)(p^2-2q)+2kqr-(6+2k)r(p^2-2q)]^2$

$ f( r)= Ar^2 +Br+C \geq 0 $ (chuẩn hóa $p=3$ )
$ A=324k^2+36k^2q^2+216kq^2+2916k-3888q+432q^2+8748-216k^2q-1620kq$
$B= 8748 -432q^3+4320q^2-126361-72kq^3-972kq+540kq^2$
$C=16q^5-144q^4+324q^3$

Xét 2 TH :
1) $ 0 \leq q \leq \dfrac{3(k+11-\sqrt{k^2+10k+49} ) }{2(k+6)} \Rightarrow B\geq 0 \text{có} C\geq 0, A\geq 0 \Rightarrow f( r) \geq 0 $

2)$ \dfrac{3(k+11-\sqrt{k^2+10k+49} ) }{2(k+6)} \leq q \leq 3 $
$ \Delta = B^2 -4AC = -144(q-3)^2(2q-9)^2(48q^3+24kq^3+4k^2q^3-144kq^2-468q^2-9k^2q^2+162kq+1296q-719) $

Từ đây suy ra $ k_{max} = 3\sqrt[3]{4} - 2$

Ví dụ 6 :
Tìm hằng số tốt nhất cho bất đẳng thức sau đúng với mọi số thực không âm a,b,c :
$ \dfrac{a^2}{b} + \dfrac{b^2}{c} + \dfrac{c^2}{a} + k(a+b+c) \geq 3(k+1)\dfrac{a^2+b^2+c^2}{a+b+c}(6)$

GiảI :
Ta có :
$ 2\sum \limits_{cyc}\dfrac{a^2}{b} = (\sum \limits_{cyc}\dfrac{a^2}{b} + \sum \limits_{cyc}\dfrac{b^2}{a}) + (\sum \limits_{cyc}\dfrac{a^2}{b} - \sum \limits_{cyc}\dfrac{b^2}{a}) = \dfrac {\sum \limits_{cyc}a^3(b+c)}{abc} + \dfrac{(a+b+c)(a-b)(b-c)(c-a)}{abc} $

$(6) \Leftrightarrow 2(\sum \limits_{cyc}\dfrac{a^2}{b} + 2k(a+b+c) \geq 6(k+1)\dfrac{a^2+b^2+c^2}{a+b+c}$

$\Leftrightarrow \dfrac {\sum \limits_{cyc}a^3(b+c)}{abc} + 2k(a+b+c) - 6(k+1)\dfrac{a^2+b^2+c^2}{a+b+c} \geq \dfrac{(a+b+c)(a-b)(b-c)(a-c)}{abc}$

Ta chỉ cần CM trong TH : $(a-b)(b-c)(a-c) \geq 0 $
$\Leftrightarrow \dfrac{p^2q-2q^2-pr}{r} +2kp -6(k+1)\dfrac{p^2-2q}{p} \geq \dfrac{p\sqrt{p^2q^2+18pqr-27r^2-4q^3-4p^3r}}{r} $

$ \Leftrightarrow p^4(p^2q^2+18pqr-27r^2-4q^3-4p^3r) \leq [(p^2q-2q^2-pr)p +2kp^2r-6(k+1)r(p^2-2q)]^2 $

$\Leftrightarrow f( r)= Ar^2 +Br+C \geq 0 $ ( Chuẩn hóa $p=3$ )
$ A= 288kq^2+144k^2q^2+1296k^2-1512q+4536k-2376kq-846k^2q+144q^2+6165$
$B=8748 -1944kq+1080kq^2-144kq^3-7776q+1404q^2-144q^3$
$C=36q^4$

Phương trình $B=0$ có nghiệm $ q \in [0,3]$:
$q_0 = \dfrac{1}{4(1+k)}(\sqrt[3]{M} + \dfrac{28k^2-100k-119}{\sqrt[3]{M}} + 10k + 3 ) $
Với :
$ M= -1475-2382k-960k^2-80k^3+36\sqrt{N} +36k\sqrt{N}$
$ N= -12k^4+324k63-63k^2+2742k+2979$

Xét 2 TH :
1)$ 0 \leq q \leq q_0 \Rightarrow B \geq 0 ; C \geq 0 \Rightarrow f( r ) \geq 0 $( Cm được $ A\geq 0 $ )

2)$q_0 \leq q \leq 3$
$ \Delta = B^2-4AC = -11644(q-3)^2(16q^3+16k^2q^3+32kq^3-252kq^2-189q^2-36k^2q^2+324kq+810q-729) $

Từ đây suy ra $ k_{max} \approx 1,5855400068$

Ví dụ 7 :
Tìm hằng số k tốt nhất sao cho bđt sau đúng với mọi $ a,b,c \geq 0 $
$\dfrac{a}{b} +\dfrac{b}{c} +\dfrac{c}{a} +k \geq \dfrac{(9+3k)(a^2+b^2+c^2)}{(a+b+c)^2}$ (7)

Giải :
Tương tự như VD 5 :
(7) $\Leftrightarrow 2\sum \limits_{cyc} \dfrac{a}{b} + 2k \geq \dfrac{6(3+k)(a^2+b^2+c^2)}{(a+b+c)^2}$

$\Leftrightarrow \dfrac{\sum \limits_{sym}a^2(b+c)}{abc} + 2k - \dfrac{6(3+k)(a^2+b^2+c^2)}{(a+b+c)^2} \geq \dfrac{(a-b)(b-c)(a-c)}{abc}$


Ta chỉ cần CM trong TH : $(a-b)(b-c)(a-c) \geq 0 $
$\Leftrightarrow \dfrac{pq-3r}{r}+ 2k - \dfrac{6(3+k)(p^2-2q)}{p^2} \geq \dfrac{\sqrt{p^2q^2+18pqr-27r^2-4q^3-4p^3r}}{r}$

$\Leftrightarrow (p^2q^2+18pqr-27r^2-4q^3-4p^3r)p^4 \leq [(pq-3r)p^2+2kp^2r-6(3+k)(p^2-2q)r]^2 $

$\Leftrightarrow f( r)= Ar^2 +Br+C \geq 0 $ ( Chuẩn hóa $p=3$ )
$ A= 144k^2q^2 +864kq^2+1296k^2-13608q+13608k+1296q^2-864k^2q-7128kq+37098$
$ B= 648kq^2-1944kq-14580+1944q^2+8748$
$ C= 324q^3$
Xét 2 TH :
1)$0 \leq q \leq \dfrac{3(15+2k-\sqrt{153+36k+4k^2})}{4(3+k)} \Rightarrow B\geq 0 $ Ta Cm được $ A\geq 0 , C\geq 0 \Rightarrow f( r ) \ge 0$

2)$ \dfrac{3(15+2k-\sqrt{153+36k+4k^2})}{4(3+k)} \geq q \geq 3 $
$ \Delta = B^2-4AC = -11664(q-3)^2(16k^2q^3+96kq^3+144q^3-36k^2q^2-972q^2-432kq^2+324kq+1944q-729) $
Từ đây suy ra $ k_{mac}= 3\sqrt[3]{2} -3$

Ví dụ 8:Tìm hằng số $k$ tốt nhất sao cho bất đẳng thức sau đúng với mọi số thực dương $a,b,c$:
$\dfrac{a}{b}+\dfrac{b}{c}+\dfrac{c}{a} \ge \dfrac{k(a^2+b^2+c^2)}{ab+bc+ca}-k+3 :Rightarrow$

Lời giải:
Tương tự ví dụ 5:
$2\sum_{cyc} \dfrac{a}{b} \ge \dfrac{2k\sum_{sym} a^2}{\sum_{sym} bc} -2k+6 \\ \Leftrightarrow \dfrac{\sum_{sym} a^2(b+c)}{abc}-\dfrac{2\sum_{sym} a^2}{\sum_{sym} bc}+2k-6 \ge \dfrac{(a-b)(b-c)(a-c)}{abc}$
Ta chỉ cần chứng minh trong trường hợp $(a-b)(b-c)(a-c) \ge 0$
$\Leftrightarrow \dfrac{pq-3r}{r}-\dfrac{2k(p^2-2q)}{q}+2k-6 \ge \dfrac{\sqrt{p^2q^2+18pqr-27r^2-4q^3-4p^3r}}{r} \\ \Leftrightarrow q^2(p^2q^2+18pqr-27r^2-4q^3-4p^3r) \le \left[(pq-3r)q-2kr(p^2-2q)+(2k-6)rq\right]^2$
$\Leftrightarrow f( r )=Ar^2+Br+C \ge 0$ (chuẩn hóa $p=3$)
$A=36k^2q^2-216k^2q+324k^2-108kq^2+324kq+108q^2$
$B=36kq^3-108q^3+108q^2-108kq^2$
$C=4q^5$
Từ đây suy ra $k_{max} =1$

Bài viết đã được chỉnh sửa nội dung bởi 10maths_tp0609: 27-09-2007 - 14:33

Zarai "từ cấm"a XIII

#2
Harry Potter

Harry Potter

    Kẻ Được Chọn

  • Hiệp sỹ
  • 286 Bài viết
Ừ cái này khá hay đấy . Các bạn làm mình nhớ lại một bào toán của mathboy :
Tìm k max sao cho :
$(a+b+c)(\dfrac{1}{a}+\dfrac{1}{b}+\dfrac{1}{c})\geq 9+k.\dfrac{MAX[(a-b)^{2};(b-c)^{2}+(c-a)^{2}]}{(a+b+c)^{2}}$

We will always have STEM with us. Some things will drop out of the public eye and will go away, but there will always be science, engineering, and technology. And there will always, always be mathematics.
 


#3
vo thanh van

vo thanh van

    Võ Thành Văn

  • Hiệp sỹ
  • 1197 Bài viết

Ừ cái này khá hay đấy . Các bạn làm mình nhớ lại một bào toán của mathboy :
Tìm k max sao cho :
$(a+b+c)(\dfrac{1}{a}+\dfrac{1}{b}+\dfrac{1}{c})\geq 9+k.\dfrac{MAX[(a-b)^{2};(b-c)^{2}+(c-a)^{2}]}{(a+b+c)^{2}}$

Mathboy chẳng phải là bạn đó sao :Rightarrow
Bài này có lời giải như sau( của tác giả :Rightarrow )Nếu rảnh thì mình sẽ post cách khác
Lời giải :
Không mất tính tổng quát GS : $a\geq b\geq b\geq c$
Bdt tương đương với : $\sum\dfrac{(a-b)^{2}}{ab} \geq k\dfrac{(a-c)^{2}}{a+b+c} $
Đặt $x=b-c;y=a-b;z=c$
Xét $x> 0$ vì hai vế của bđt là đồng bậc nên hoàn toàn có thể giả sử $x=1$
đưa bđt thành : $\dfrac{[z(y^{2}+(y+1)^{2}+1)+(y+1)(y+2)](3z+y+2)^{2}}{z(z+1)(z+y+1)(y+1)^{2}}\geq k$:Rightarrow
Cho z tiến tới vô cùng thì ta có : $k\leq \dfrac{[y^{2}+1+(y+1)^{2}]9}{(b+1)^{2}}=M(y)$
Dễ dàng CM : $M(y)\geq \dfrac{27}{2}$ Vậy $k\leq \dfrac{27}{2}$
Ta chỉ cần CMR : Với $k=\dfrac{27}{2}$ thì bất đẳng thức sẽ thỏa mãn
Thật vậy để cm điều này các bạn đưa (**)(lúc này $k\dfrac{27}{2}$ Thành một phương trình bậc 3 ẩn z rồi Cm các hệ số $\geq 0$ điều này khá đơn giản
Vậy ta có $k(max)=\dfrac{27}{2}$
Khi $x=0$ các bạn có thể dễ dàng CM $k(max)=16$
Nhưng xin mọi người đừng cho rằng điều này mâu thuẫn với kết luận ở trên
Vì $k(max)=27$ sẽ thỏa mãn Với mọi bộ (a:b:c) còn $k(max)=16$ chỉ được khi có hai biến bằng nhau
Quy ẩn giang hồ

#4
Harry Potter

Harry Potter

    Kẻ Được Chọn

  • Hiệp sỹ
  • 286 Bài viết
Ừ bài này có thể giải bằng dồn biến . Nhưng tớ không phải mathboy :D .Harry Potter đúng là nick của mathboy , nhưng Harry Potter hiện nay thì không phải. Tớ học dốt lắm , không phải là mathboy đâu

We will always have STEM with us. Some things will drop out of the public eye and will go away, but there will always be science, engineering, and technology. And there will always, always be mathematics.
 


#5
zaizai

zaizai

    Tiến sĩ diễn đàn toán

  • Thành viên
  • 1380 Bài viết
quả là 1 bài viết rất hay và ý tưởng đối xứng hoá rất thú vị. Theo mình thì nhược điểm lớn nhất là mức độ tính toán ko nhỏ và một số bài quá khủng hoảng và nếu quá khủng hoảng như vậy thà dùng phép đặt $a=min(a,b,c),b=a+p,c=a+1$ ngay sau khi qui về có chứa đại lượng $(a-b)(b-c)(a-c)$ có khi còn đỡ suy nghĩ hơn :D Tuy nhiên phải công nhận rằng nó không hề yếu. Rất vui vì p,q,r tiếp tục được phát triển lên 1 tầm cao mới :D

#6
vịt con

vịt con

    Hạ sĩ

  • Thành viên
  • 66 Bài viết

thà dùng phép đặt $a=min(a,b,c),b=a+p,c=a+1$ ngay sau khi qui về có chứa đại lượng $(a-b)(b-c)(a-c)$ có khi còn đỡ suy nghĩ hơn

Anh nói rõ thêm đi

#7
zaizai

zaizai

    Tiến sĩ diễn đàn toán

  • Thành viên
  • 1380 Bài viết

Anh nói rõ thêm đi

Uh tư tưởng của nó cũng đơn giản thôi và theo anh thì những bài trên ko khó đối với nó và các lời giải quá khủng hoảng như trên của 10math (theo mình nghĩ có sự can thiệp của maple :D) thì thật sự cái trò mà anh nói cũng ko khác là mấy và đơn giản hơn về mặt ý tưởng. Tư tưởng là quy về xét hàm theo biến $a$ với các hệ số là là các đa thức $ g(p)$ thường thì nó làm việc khá tốt.
Có thể minh hoạ bằng 1 bài ngày xưa nổi đình nổi đám bên MnF (ngày xưa nó là 1 bài unsolve bên MnF còn bây giờ thì chắc là khác xa rồi :D)
Cho $a,b,c$ là các số thực dương thoả mãn $abc=1.$ Chứng minh rằng:

$\dfrac{a}{b^4+2}+\dfrac{b}{c^4+2}+\dfrac{c}{a^4+2}\ge 1$

Với bài này thì cách trên của 10maths vẫn works nhưng mà xem ra cũng khủng hoảng lắm :D Xin lỗi vì thấy anh Tanpham dùng từ "khủng hoảng" cũng thấy hay hay nên học theo :D

#8
dduclam

dduclam

    Sĩ quan

  • Thành viên
  • 364 Bài viết
Mình có mấy bài này nhờ mọi người gúp đỡ:
Bài1: Cho a,b,c>0.CMR: $\dfrac{\sqrt[3]{a^3+b^3+c^3}}{3abc}\ge \dfrac{(\sqrt{a}+ \sqrt{b}+ \sqrt{c}}{(ab+bc+ca)}$
Bai2:Cho $a,b,c>=0$ và a2+b2+c2=3.CMR: $8(a+b+c)^2\ge 9(a^2+bc)(b^2+ca)(c^2+ab)$.
Bài3:Cho $a,b,c\ge 0$.CMR: $\dfrac{a}{(b+c)^4}+\dfrac{b}{(c+a)^4}+\dfrac{c}{(a+b)^4}\ge \dfrac{81}{(16(a+b+c)^3}$.
Đều là BĐT đối xứng cả.

Bài viết đã được chỉnh sửa nội dung bởi zaizai: 22-10-2007 - 00:30

Sống trên đời cần có một tấm lòng
để làm gì em biết không?
để gió cuốn đi...

Khi ước mơ đủ lớn, mọi thứ khác chỉ là vặt vãnh

#9
10maths_tp0609

10maths_tp0609

    Zarai Nakeda XIII

  • Thành viên
  • 218 Bài viết

Uh tư tưởng của nó cũng đơn giản thôi và theo anh thì những bài trên ko khó đối với nó và các lời giải quá khủng hoảng như trên của 10math (theo mình nghĩ có sự can thiệp của maple :D) thì thật sự cái trò mà anh nói cũng ko khác là mấy và đơn giản hơn về mặt ý tưởng. Tư tưởng là quy về xét hàm theo biến $a$ với các hệ số là là các đa thức $ g(p)$ thường thì nó làm việc khá tốt.
Có thể minh hoạ bằng 1 bài ngày xưa nổi đình nổi đám bên MnF (ngày xưa nó là 1 bài unsolve bên MnF còn bây giờ thì chắc là khác xa rồi :D)
Cho $a,b,c$ là các số thực dương thoả mãn $abc=1.$ Chứng minh rằng:

$\dfrac{a}{b^4+2}+\dfrac{b}{c^4+2}+\dfrac{c}{a^4+2}\ge 1$

Với bài này thì cách trên của 10maths vẫn works nhưng mà xem ra cũng khủng hoảng lắm :D Xin lỗi vì thấy anh Tanpham dùng từ "khủng hoảng" cũng thấy hay hay nên học theo :D


hi, 2 cái này thực ra khác nhau nhiều lắm. Như ví dụ 4, nếu đặt theo cách như zai nói, g sử $c=min$, đặt $a=c+x,b=c+1$ thì bđt trở thành:
$-kc^3x^2+kcx^2-kxc+c+2c^2+x+kc^3x-kx^3c-kx^3c^2+cx+3xc^2+4cx^2-3c^3x+3c^3x^2+3c^3+x^3+2c^2x^3-x^3c+x^4c-kc^3-kc^2 \ge 0$
Đây là hàm bậc 3 của $c-->> $khử $c $ đi khó hơn nhiều so với cách trên khi bậc của $r$ chỉ có 2.

Hơn nữa có sự khá biệt rõ ràng nữa là với các bài chứa căn, ví dụ bài:
Tìm $k$ tốt nhất:
$\dfrac{a}{b}+\dfrac{b}{c}+\dfrac{c}{a}+k \ge (3+k)\left(\dfrac{a^2+b^2+c^2}{ab+bc+ca}\right)^{\dfrac{2}{3}}$
Bài này dùng như cách trên vẫn solve đơn giản còn nếu đặt như zai nói thì không biết phải tiến hành tiếp tục như thế nào.

====================================
p/s: đây là file 1maths đã tổng hợp, định trans sang TV nhưng cũng còn nhiều lỗi và cũng chưa hoàn thành nên để khi hoàn tất chỉnh sửa sau vậy, mọi người xem tạm nhé.

File gửi kèm


Bài viết đã được chỉnh sửa nội dung bởi 10maths_tp0609: 18-10-2007 - 23:15

Zarai "từ cấm"a XIII

#10
zaizai

zaizai

    Tiến sĩ diễn đàn toán

  • Thành viên
  • 1380 Bài viết
ví dụ 4 là bdt vasc mà chả lẽ nó ra tới bậc 3 :phi thực ra bậc 3 chắc cũng ko mấy khó khăn (thường thì các biểu thức $g(p)$ ko âm còn ko thì dùng chút đạo hàm nữa :D nhưng còn căn thì đúng là cần xem xét nhiều hơn nhưng chắc là vẫn work (cùng lắm thì khử căn và trâu bò ra :phi). Ko có thời gian làm mấy trò cồng kềnh này nên thông cảm nhá :D
Nói chung thì ko thể phủ nhận những kết quả của 10maths là rất mạnh nhưng mà thú thật là mình ko thích lắm vì nó cồng kềnh (maple đc dùng nhìu lần :D), p,q,r cho bdt đối xứng thường khá đẹp và gọn gàng :D nhưng tất nhiên vì đây là hoán vị nên khác nhau cũng là điều tất nhiên :D thôi chúc 10maths tiếp tục hoàn thiện pp này, tớ phải học những thứ khác đây :D

Bài viết đã được chỉnh sửa nội dung bởi zaizai: 19-10-2007 - 19:43


#11
10maths_tp0609

10maths_tp0609

    Zarai Nakeda XIII

  • Thành viên
  • 218 Bài viết

ví dụ 4 là bdt vasc mà chả lẽ nó ra tới bậc 3 :D thực ra bậc 3 chắc cũng ko mấy khó khăn (thường thì các biểu thức $g(p)$ ko âm còn ko thì dùng chút đạo hàm nữa :D nhưng còn căn thì đúng là cần xem xét nhiều hơn nhưng chắc là vẫn work (cùng lắm thì khử căn và trâu bò ra :phi). Ko có thời gian làm mấy trò cồng kềnh này nên thông cảm nhá :phi
Nói chung thì ko thể phủ nhận những kết quả của 10maths là rất mạnh nhưng mà thú thật là mình ko thích lắm vì nó cồng kềnh (maple đc dùng nhìu lần :D), p,q,r cho bdt đối xứng thường khá đẹp và gọn gàng :D nhưng tất nhiên vì đây là hoán vị nên khác nhau cũng là điều tất nhiên :D thôi chúc 10maths tiếp tục hoàn thiện pp này, tớ phải học những thứ khác đây :D



Nhầm chút, thực ra đoạn trên là ví dụ 5, ví dụ 4 thì lời giải như cách của zai nói quá nổi tiếng rồi.

p/s: sorry mọi người, trong file thứ 2 mình up lên có một lỗi typing sai ngay đoạn đầu tiên, thực ra là $\delta \le 0$ thay vì $\delta \ge 0$ như trong bài viết

Bài viết đã được chỉnh sửa nội dung bởi 10maths_tp0609: 20-10-2007 - 17:04

Zarai "từ cấm"a XIII

#12
vịt con

vịt con

    Hạ sĩ

  • Thành viên
  • 66 Bài viết
2 anh nói vậy thì em cũng chịu thua,không ai giải thích cho em hiểu hết hu..hu :sum:limits_{i=1}^{n}

#13
vipCDa1

vipCDa1

    Hạ sĩ

  • Thành viên
  • 58 Bài viết
Những cái phân tích của 10math ko làm bằng tay được phải nhờ máy mới ra hay sao
TRẦN CÔNG DIÊU
NHÓM PROMATH
DIỄN ĐÀN http://ant.edu.ms

#14
zaizai

zaizai

    Tiến sĩ diễn đàn toán

  • Thành viên
  • 1380 Bài viết

Những cái phân tích của 10math ko làm bằng tay được phải nhờ máy mới ra hay sao

Ko hắn, tất nhiên là vẫn có thể dùng tay nhưng nó nặng nề lắm chứ chẳng nhẹ nhàng mấy :sum:limits_{i=1}^{n} còn về câu hỏi của vitcon như mình đã nói thực ra nó chỉ là thế vào và nhóm lại theo biến mà ta đã chọn là nhỏ hoặc lớn nhất trong 3 biến thôi. 1 đa thức có dạng:
$\sum a^n\times g(p)\ge 0$
Công việc bây giờ là xét hàm theo biến a và hệ số là g(p). Ví dụ luôn là bài 1 chẳng hạn:
Đồng bậc sẽ có:
$27(a^2b+b^2c+c^2a)\le 4(a+b+c)^3$
Sau đó ta giả sử $a=min[a,b,c]$ sẽ có $b=a+p,c=a+1,p\ge 0$
Thế hàm $f(a,b,c)$ thành xét hàm $f(a,a+p,a+1)$
Sẽ có:
$f(a,a+p,a+1)=27a^3+(27p+27)a^2+(9p^2+18p+9)a+4p^3-15p^2+12p+4$
Rõ ràng: $4p^3-15p^2+12p+4$ đạt min tại $p=2$ và lớn hơn $0$
Chỉ vậy thôi :sum

Bài viết đã được chỉnh sửa nội dung bởi zaizai: 22-10-2007 - 00:17


#15
zaizai

zaizai

    Tiến sĩ diễn đàn toán

  • Thành viên
  • 1380 Bài viết
theo em 2 bài đầu dùng $p,q,r$ còn bài sau chuẩn hoá $a+b+c=3$ thì bài toán chỉ còn là bdt 1 biến :sum:limits_{i=1}^{n}

#16
vipCDa1

vipCDa1

    Hạ sĩ

  • Thành viên
  • 58 Bài viết
Bạn thấy đấy cái này ngán thật nhưng rất đơn giản về mặt ý tưởng nhưng đòi hỏi khéo léo
TRẦN CÔNG DIÊU
NHÓM PROMATH
DIỄN ĐÀN http://ant.edu.ms

#17
vipCDa1

vipCDa1

    Hạ sĩ

  • Thành viên
  • 58 Bài viết
Thử bài này nè 10math và zaizai

[Công Diêu] Cho $a,b,c>0$ thỏa $abc=1$. Chứng minh:

$b^2c^4+a^2b^4+c^2a^4+3k\geq{(k+1)(\dfrac{a}{c}+\dfrac{b}{a}+\dfrac{c}{b})$
khá hay đấy chứ
CŨNG KHÔNG CÓ GÌ QUÁ MỚI MẼ VÀ KHÓ KHĂN

Bài viết đã được chỉnh sửa nội dung bởi anh3rau: 27-10-2007 - 20:28

TRẦN CÔNG DIÊU
NHÓM PROMATH
DIỄN ĐÀN http://ant.edu.ms

#18
vipCDa1

vipCDa1

    Hạ sĩ

  • Thành viên
  • 58 Bài viết
Các bài 10math giải phần trước chỉ đơn thuần là biến đổi còn phần sau là rất khó phải đánh giá nhiều mà thực chất là có thể sai rất dễ dẫn đến nản, có một cách khá tốt để phá vỡ sự cồng kềnh đó
TRẦN CÔNG DIÊU
NHÓM PROMATH
DIỄN ĐÀN http://ant.edu.ms

#19
zaizai

zaizai

    Tiến sĩ diễn đàn toán

  • Thành viên
  • 1380 Bài viết
ko nên chỉ nói úp mở như vậy :) mong bạn post lên để mọi người cùng tham khảo :P

#20
PrT

PrT

    Thượng sĩ

  • Thành viên
  • 218 Bài viết

Ko hắn, tất nhiên là vẫn có thể dùng tay nhưng nó nặng nề lắm chứ chẳng nhẹ nhàng mấy :P còn về câu hỏi của vitcon như mình đã nói thực ra nó chỉ là thế vào và nhóm lại theo biến mà ta đã chọn là nhỏ hoặc lớn nhất trong 3 biến thôi. 1 đa thức có dạng:
$\sum a^n\times g(p)\ge 0$
Công việc bây giờ là xét hàm theo biến a và hệ số là g(p). Ví dụ luôn là bài 1 chẳng hạn:
Đồng bậc sẽ có:
$27(a^2b+b^2c+c^2a)\le 4(a+b+c)^3$
Sau đó ta giả sử $a=min[a,b,c]$ sẽ có $b=a+p,c=a+1,p\ge 0$
Thế hàm $f(a,b,c)$ thành xét hàm $f(a,a+p,a+1)$
Sẽ có:
$f(a,a+p,a+1)=27a^3+(27p+27)a^2+(9p^2+18p+9)a+4p^3-15p^2+12p+4$
Rõ ràng: $4p^3-15p^2+12p+4$ đạt min tại $p=2$ và lớn hơn $0$
Chỉ vậy thôi :)

Thực sự chưa hiểu lắm mấy cái này tại sao lại xét c=a+1 chứ như thế thì dấu = khi nào ?
Nới thật nếu đã áp dụng thế sao ko dùng luông ABC đi (mặc dù ko phải lúc nào cũng đựoc nhưng công nhận pp đó mạnh )
God does Mathematics.




1 người đang xem chủ đề

0 thành viên, 1 khách, 0 thành viên ẩn danh